Add solutions for Serway and Jewett v8's chapter 31.
[course.git] / latex / problems / Serway_and_Jewett_8 / problem31.45.tex
1 \begin{problem*}{31.45}
2 A circular coil enclosing an area $A=0.0100\U{m$^2$}$ is made of $200$
3 turns of copper wire as shown in Figure~P31.45.  Initially, a uniform
4 magnetic field of magnitude $B=1.10\U{T}$ points upward in a direction
5 perpendicular to the plane of the coil.  The direction of the field
6 then reverses in a time interval $\Delta t$.  Determine how much
7 charge enters one end of the resistor during this time interval if
8 $R=5.00\U{Ohm}$.
9 \begin{center}
10 % B ^ ^ ^
11 %   | | |  (from back of coil)
12 %   +-+-+---+
13 %   +-+-+   Z R
14 %   +-+-+   Z
15 %   +-+-+---+
16 %   | | |  (from font of coil)
17 \begin{asy}
18 import Mechanics;
19 import ElectroMag;
20 import Circ;
21
22 real dx = 5mm;
23 real r = 5mm;
24 int n = 10;      // number of coil loops
25 real tense = 8;  // increasing tension flattens the coil loops
26
27 MultiTerminal R = resistor(dir=90, Label("$R$", align=E));
28 real rlen = R.terminal[1].y - R.terminal[0].y;
29 real xr = R.center.x - dx;
30 real xl = xr - 2*r;
31 real dy = rlen / n;
32 for (int i = 0; i < n; i += 1) {  // back sides of coil
33   pair dout = S;
34   if (i == n-1) {
35     dout = E;
36   }
37   real y0 = R.terminal[0].y + i * dy;
38   draw((xl, y0 + dy/2){N}..tension tense ..{dout}(xr, y0 + dy));
39 }
40 Vector B = BField(dir=90);
41 B.outline += linewidth(0.5mm);
42 vector_field(((xl+xr)/2, R.center.y+rlen/3), width=2*r, height=2*rlen, v=B);
43 for (int i = 0; i < n; i += 1) {  // front sides of coil
44   pair din = S;
45   if (i == 0) {
46     din = W;
47   }
48   real y0 = R.terminal[0].y + i * dy;
49   draw((xr, y0){din}..tension tense ..{N}(xl, y0 + dy/2));
50 }
51 draw((xr, R.terminal[0].y) -- R.terminal[0]);
52 draw((xr, R.terminal[1].y) -- R.terminal[1]);
53 \end{asy}
54 \end{center}
55 \end{problem*}
56
57 \begin{solution}
58 Picking up as the positive flux direction, the induced \EMF\ is
59 \begin{equation}
60   \EMF = -\deriv{t}{\Phi_B} = -\deriv{t}{NAB} = -NA\deriv{t}{B}
61     = -NA\frac{-2B}{\Delta t} = \frac{2NAB}{\Delta t} \;.
62 \end{equation}
63 This \EMF\ drives a current through the resistor
64 \begin{align}
65   0 &= \EMF - IR \\
66   I &= \frac{\EMF}{R} = \frac{-2NAB}{R\Delta t} \;.
67 \end{align}
68 Current is defined as the charge passing through a cross section of
69 your circuit per unit time, so the charge entering one end of the
70 resistor is
71 \begin{equation}
72   \Delta q = \frac{\Delta q}{\Delta t} \Delta t = I \Delta t
73     = \frac{2NAB}{R} = \ans{0.880\U{C}} \;.
74 \end{equation}
75 \end{solution}